Triangle mother

Geometry Level 4

Triangle A B C ABC lies in the cartesian plane and has an area of 70. The coordinates of B B and C C are ( 12 , 19 ) (12,19) and ( 23 , 20 ) (23,20) , respectively, and the coordinates of A A are ( p , q ) (p,q) . The line containing the median to side B C BC has slope 5 -5 . Find the largest possible value of p + q p+q .


The answer is 47.

This section requires Javascript.
You are seeing this because something didn't load right. We suggest you, (a) try refreshing the page, (b) enabling javascript if it is disabled on your browser and, finally, (c) loading the non-javascript version of this page . We're sorry about the hassle.

1 solution

Relevant Wikis

Gauss's Shoelace Formula

Mid Point of Line Segment

Equation of The Line

I think that my solution isn't the most simplest one, but I enjoyed so much solving this way!

The Gauss's Shoelace Formula allows us to find a area of a simple polygon whose vertices are in the Cartesian Plane by using matrices. Observe the following image:

We multiply two numbers connected by a red slash and sum up the products. We obtain 19 p + 240 + 23 q 19p+240+23q . Then we multiply two numbers connected by a green slash and sum up the products. We obtain 12 q + 437 + 20 p 12q+437+20p .

The modulus of the difference between these two results is 2 times the area of Triangle A B C ABC which is 2 × 70 = 140 2\times70=140 . Then we have ( 19 p + 240 + 23 q ) ( 12 q + 437 + 20 p ) = p + 197 11 q = 140 |(19p+240+23q)-(12q+437+20p)|=|p+197-11q|=140 .

Now we can get another equation by using the information about the median.

The median of a triangle is the segment that connects a vertex to the midpoint of its opposite side. Then we need to find the midpoint of the segment B C BC . The coordinates of the segment that connects points ( x 1 , y 1 ) (x_1,y_1) and ( x 2 , y 2 ) (x_2,y_2) is ( x 1 + x 2 2 , y 1 + y 2 2 ) \left(\frac{x_1+x_2}{2},\frac{y_1+y_2}{2}\right) .

Then, the coordinates of the midpoint M M of the segment that connects points B ( 12 , 19 ) B(12,19) and C ( 23 , 20 ) C(23,20) is ( 12 + 23 2 , 19 + 20 2 ) = M ( 17.5 , 19.5 ) \left(\frac{12+23}{2},\frac{19+20}{2}\right)=M(17.5,19.5)

Now we need to find the equation of the line that passes trough points M ( 17.5 , 19.5 ) M(17.5,19.5) and A ( p , q ) A(p, q) . To do so I will use my favorite method that use the following formula to find the general equation of a line. This formula can be obtained by using similar triangles:

Equation of the line that passes trough points ( x 1 , y 1 ) (x_1,y_1) and ( x 2 , y 2 ) (x_2,y_2) is ( x x 1 x 1 x 2 = y y 1 y 1 y 2 ) \left(\frac{x-x_1}{x_1-x_2}=\frac{y-y_1}{y_1-y_2}\right) . Solving for points A ( p , q ) A(p, q) and M ( 17.5 , 19.5 ) M(17.5,19.5) :

( x p p 17.5 = y q q 19.5 ) \left(\frac{x-p}{p-17.5}=\frac{y-q}{q-19.5}\right)

( x p ) ( q 19.5 ) = ( y q ) ( 9 17.5 ) (x-p)(q-19.5)=(y-q)(9-17.5)

x q 19.5 x p q + 19.5 p = y p 17.5 y p q + 17.5 q xq-19.5x-pq+19.5p=yp-17.5y-pq+17.5q

x q 19.5 x + 19.5 p = y p 17.5 y + 17.5 q xq-19.5x+19.5p=yp-17.5y+17.5q

Manipulation the equation to get the reduced equation in the form m a + b = y ma+b=y where m m is the slope of the line:

x q 19 , 5 x + 19 , 5 p = y p 17 , 5 y + 17 , 5 q xq-19,5x+19,5p=yp-17,5y+17,5q

x q 19.5 x + 19.5 p 17.5 q = y p 17.5 y xq-19.5x+19.5p-17.5q=yp-17.5y

x ( q 19.5 ) + 19.5 p 17.5 q = y ( p 17.5 ) x(q-19.5)+19.5p-17.5q=y(p-17.5)

( q 19.5 ) ( p 17.5 ) x + ( 19.5 p 17.5 q ) ( p 17.5 ) = y \frac{(q-19.5)}{(p-17.5)}x+\frac{(19.5p-17.5q)}{(p-17.5)}=y

We can see in this equation by compared to the reduced one that our m = ( q 19.5 ) ( p 17.5 ) m=\frac{(q-19.5)}{(p-17.5)} . In this case the slope is equal to 5 -5 . Then we have ( q 19.5 ) ( p 17.5 ) = 5 \frac{(q-19.5)}{(p-17.5)}=-5

Now we have the following system of equations:

{ q 19.5 p 17.5 = 5 p + 197 11 q = 140 \begin{cases} \frac { q-19.5 }{ p-17.5 }=-5 \\ |p+197-11q|=140 \end{cases}

For the second equation we have two cases: (i) p + 197 11 q = 140 p+197-11q=140 and (ii) p + 197 11 q = 140 p+197-11q=-140 . Solving for each case:

Case (i)

p + 197 11 q = 140 p+197-11q=140

p = 11 q 57 p=11q-57

Substituting in the first equation:

q 19.5 p 17.5 = 5 \frac { q-19.5 }{ p-17.5 }=-5

q 19.5 11 q 57 17.5 = 5 \frac { q-19.5}{ 11q-57-17.5 }=-5

q 19.5 11 q 74 , 5 = 5 \frac { q-19.5 }{ 11q-74,5}=-5

q 19.5 = 5 ( 11 q 74 , 5 ) q-19.5 =-5(11q-74,5)

q 19.5 = ( 55 q + 372 , 5 ) q-19.5 =(-55q+372,5)

56 q = ( 392 56q =(392

q = 7 q=7

p = 11 ( 7 ) 57 = 20 p=11\cdot(7)-57=20

Then we have p = 20 p=20 and q = 7 q=7 . Then p + q = 20 + 7 = 27 p+q=20+7=27 in this case.

Case (ii)

p + 197 11 q = 140 p+197-11q=-140

p = 11 q 377 p=11q-377

Substituting in the first equation:

q 19.5 p 17.5 = 5 \frac { q-19.5 }{ p-17.5 }=-5

q 19.5 11 q 337 17.5 = 5 \frac { q-19.5}{ 11q-337-17.5 }=-5

q 19.5 11 q 354 , 5 = 5 \frac { q-19.5 }{ 11q-354,5}=-5

q 19.5 = 5 ( 11 q 354 , 5 ) q-19.5 =-5(11q-354,5)

q 19.5 = ( 55 q + 1792 ) q-19.5 =(-55q+1792)

56 q = ( 1792 56q =(1792

q = 32 q=32

p = 11 ( 32 ) 57 = 15 p=11\cdot(32)-57=15

Then we have p = 15 p=15 and q = 32 q=32 .

Then p + q p+q will be 15 + 32 = 47 15+32=47 in this case.

47 > 27 47>27

Hence the largest possible value for p + q p+q is 47 \boxed{47}

0 pending reports

×

Problem Loading...

Note Loading...

Set Loading...